Warum sind die Lagrange-Punkte L1L1L_1, L2L2L_2 und L3L3L_3 instabil?

Warum sind die Lagrange-Punkte L 1 , L 2 & L 3 instabil? Ich mache eine Physikpräsentation vor meiner Klasse und ich bestätige nur, dass dies richtig ist, und wenn nicht, könnte jemand eine Erklärung geben.

F = M v 2 / R Und v = 2 π / T

Deshalb:

F = 4 π 2 M R / T 2

F 1 / T 2 während R ist konstant.

Also wenn R nicht mehr am selben Punkt wie der Lagrange-Punkt ist, dann ändert sich die Umlaufdauer und das System fällt aus der Position.

Antworten (3)

I) OP fragt (v5):

Warum sind die Lagrange-Punkte L 1 , L 2 & L 3 instabil?

Eigentlich ist das eine großartige und nicht triviale Frage. Man sollte keine Schlussfolgerungen nur auf der Grundlage des effektiven Potenzials ziehen v (bestehend aus der Schwerkraft und dem Zentrifugalpotential ) allein, weil die Coriolis-Kraft wichtig sein kann.

Zum Beispiel die globalen Höchstpunkte L 4 & L 5 von v (was naiverweise die letzten Orte wären, an denen man einen stabilen Gleichgewichtspunkt vermuten würde), sind manchmal stabil, vgl. zB dieser Phys.SE Beitrag!

II) Analysieren wir hier die Stabilität eines beliebigen Punktes mit kleiner Anfangsgeschwindigkeit v 0 in der Orbitalebene 1 R 2 C in Gegenwart der Coriolis-Kraft. Lassen Sie uns Koordinaten auswählen z C so dass die Ausgangsposition im Ursprung liegt z = 0 .

Beginnen wir mit der Untersuchung kritischer/stationärer Punkte , dh Lagrange-Punkte. Wir verwenden den folgenden Satz, der in Lit. erwähnt wird. 1:

Satz. Gegeben ein Hessisch H für ein wirksames Potenzial v : R 2 R an einem Lagrange-Punkt. Lassen Ω sei die Winkelgeschwindigkeit. Die folgenden 3 Bedingungen sind notwendige & hinreichende Bedingungen, damit der Lagrange-Punkt stabil ist:

  1. C   :=   det ( H )     0.

  2. B   :=   T R ( H ) + 4 Ω 2     0.

  3. D   :=   B 2 4 C     0.

NB: Der Satz ignoriert Terme höherer Ordnung in v , was wichtig werden könnte, wenn det ( H ) = 0 .

Ein Beweis des Satzes ist in meiner Phys.SE-Antwort hier angegeben , in der auch der Fall eines lokalen Maximums erörtert wird.

  1. Fall eines lokalen Minimums 2 det ( H ) 0 T R ( H ) 0 : Es ist stabil, weil alle 3 Bedingungen erfüllt sind. Um die dritte Bedingung zu sehen, beachten Sie das

    (1) D   =   B 2 4 C   =   ( T R ( H ) + 4 Ω 2 ) 2 4 det ( H )   =   T R ( H ) 4 det ( H ) + 8 T R ( H ) Ω 2 + 16 Ω 4   =   ( H 11 H 22 ) 2 + 4 H 12 2 + 8 T R ( H ) 0 Ω 2 + 16 Ω 4   >   0.

  2. Bei starkem Sattelpunkt det ( H ) < 0 : Dies ist bei den Lagrange-Punkten der Fall L 1 , L 2 & L 3 , nach dem OP fragt. Sie ist instabil, weil sie die erste Bedingung verletzt.

III) Um eine gewisse Intuition für die obigen Schlussfolgerungen zu gewinnen, lassen Sie uns auch analysieren, was abseits der Lagrange-Punkte passiert.

  1. Fall eines unkritischen Punktes: Die spezifische Wirkkraft F = v | z = 0 0 ist ungleich Null. Dann in dem Bereich, in dem wir die spezifische Wirkkraft als konstant behandeln können, die EOM mit spezifischer Coriolis-Kraft 2 v × Ω liest 3

    (2) z ¨     2 ich Ω z ˙ Koriolis + F   D D T ( exp { 2 ich Ω T } z ˙ )     exp { 2 ich Ω T } F .
    Daher ist die Geschwindigkeit
    (3) z ˙     F 2 ich Ω + ( v 0 F 2 ich Ω ) exp { 2 ich Ω T } ,
    und die Lage ist
    (4) z     F T 2 ich Ω Drift + 1 2 ich Ω ( F 2 ich Ω v 0 ) komplexe Amplitude ( exp { 2 ich Ω T } Kreis 1 )
    (5)     v 0 T + ( F 2 ich Ω v 0 ) T 2 2 ( 1 2 ich Ω T 3 ) + Ö ( T 4 ) .
    Das Testteilchen führt eine kreisförmige Bewegung mit einer Drift entlang von Äquipotentiallinien in der Bahnebene aus. (Zum R ich G H T wenn du suchst D Ö w N H ich l l .) In jedem Fall ist das Testteilchen an einem unkritischen Punkt instabil.

    Abb. 1: Eine mögliche Hufeisenbahn entlang Äquipotentiallinien.

Die obige Beschreibung gibt einen Einblick, warum lokale Minima und (manchmal) lokale Maxima stabil sind, während Sattelpunkte immer instabil sind. Wenn das Testteilchen dazu neigt, entlang Äquipotentiallinien zu driften, dann endet es in der Nähe lokaler Minima und Maxima in kleinen konzentrischen Schleifen, während Äquipotentiallinien immer von Sattelpunkten weg zeigen.

Abb. 2: Ein Testpartikel neigt dazu, aufgrund der Coriolis-Kraft entlang Äquipotentiallinien zu driften.

Verweise:

  1. J. Binney & S. Tremaine, Galactic Dynamics, 2. Auflage (2008); P. 181-182.

--

1 Es könnte auch eine oszillierende Bewegung senkrecht zur Bahnebene geben, die wir ignorieren.

2 Die obige Analyse gilt für jedes effektive Potenzial v . Im eingeschränkten 3-Körper-Problem, nach dem OP fragt, das effektive Potenzial v hat kein lokales Minimum.

3 Hier wählen wir die Orientierung der Winkelgeschwindigkeit Ω gerader/linksdrehender/positiver Drehrichtung sein.

Ich finde, die intuitivste Art, über Lagrange-Punkte nachzudenken, besteht darin, die potenzielle Energie der Gravitation und eine Oberfläche zu betrachten, die das Potenzial darstellt.

Die Lagrange-Punkte existieren im Raum um zwei massive Objekte, sagen wir die Sonne und die Erde. Sie können sich beide so vorstellen, dass sie ein Gravitationspotential erzeugen, das ein bisschen wie ein Kegel oder Trichter geformt ist, der nach unten zeigt, aber mit Seiten, die ein gekrümmtes Profil bilden. Das Gravitationspotential als Funktion der Entfernung R wird von gegeben U = G M / R . Hier ist ein schönes Bild von Wikipedia 2 . Bildquelle: Von AllenMcC. - Eigene Arbeit, CC BY-SA 3.0, GravitationspotentialbrunnenGravitationspotential gut

Nun, da es zwei massive Körper gibt, gibt es zwei dieser Brunnen in der Nähe von jedem, obwohl derjenige, der auf dem massiveren Körper zentriert ist, größer sein wird. Sie können sich vorstellen, dass es zwischen den beiden Vertiefungen einen Sattelpunkt geben würde, an dem die Oberfläche flach wäre. Dies würde einen Punkt darstellen, an dem die Schwerkraft, die zu jedem Körper zieht, im Gleichgewicht ist (ich konnte kein gutes Bild von zwei Brunnen finden).

Die Sache ist, dass der kleinere Körper (sagen wir die Erde) den größeren Körper (Sonne) umkreist, oder genauer gesagt, beide umkreisen ihr gemeinsames Baryzentrum. Die Lagrange-Punkte sind von der Erde aus gesehen stationär, also müssen wir die Dinge von einem Bezugsrahmen aus betrachten, der sich mit der Erde dreht. In einem solchen rotierenden Bezugssystem müssen wir eine fiktive Zentrifugalkraft vom Rotationszentrum weg einführen. Die Addition dieser Kraft ändert die potentielle Oberfläche mit den Gravitationsbrunnen. Da die Kraft vom Zentrum weg wirkt, hätte ein Objekt in der Nähe des Zentrums ein höheres Potential als ein weiter entferntes, sodass das Potential für diese Kraft wie eine umgedrehte Schüssel aussehen würde. Wenn Sie die Auswirkungen der Schwerkraft von beiden Körpern und der Zentrifugalkraft zusammenrechnen, erhalten Sie eine etwas komplizierte effektive potenzielle Oberfläche. Hier'Äquipotential der Lagranschen Punkte

Bildquelle: Von Benutzer: cmglee - Eigene Arbeit, CC BY-SA 3.0, Lagranian Points Equipotential

Die fünf Lagrange-Punkte sind also Orte, an denen die Steigung dieser Potentialfläche Null ist. Du könntest dir vorstellen, dort zu sitzen und nicht von ihnen wegzurutschen, wenn du aufpasst. Der L 1 , L 2 , L 3 , sind alle sattelförmig, während die L 4 Und L 5 Punkte treten an lokalen Maxima auf. Sie sind wie Hügelkuppen, aber ziemlich flache Hügelkuppen.

Trotzdem ist die L 4 Und L 5 Punkte sind keine lokalen Minima, wieso sind sie stabil? Nun, im rotierenden Bezugssystem ist die Zentrifugalkraft nicht die einzige fiktive Kraft, die eingebracht werden muss. Sobald sich etwas zu bewegen beginnt , erfährt es auch eine Coriolis-Kraft senkrecht zu seiner Bewegung. Wenn sich ein Satellit von der entfernt L 4 oder L 5 Punkte ist es diese Coriolis-Kraft, die ihn zurück, wenn nicht genau zurück zum Lagrange-Punkt, dann zu einer Umlaufbahn um ihn herum schubst.

Mir ist gerade aufgefallen, dass ich das Falsche erklärt habe - warum L4 und L5 stabil sind. Ich werde bearbeiten.
Ein Papier wie Stability of Lagrange Points: James Webb Space Telescope gibt eine Analyse der Stabilität aller fünf Lagrange-Punkte, aber ich vermute, dass es über das Niveau hinausgeht, nach dem das OP gesucht hat. Gibt es eine einfachere Möglichkeit, die ursprüngliche Frage zu beantworten, als nur zu sagen, dass das effektive Potenzial an den fraglichen Punkten kein lokales Minimum ist und die Erklärung der Coriolis-Kraft, die wir für L4 und L5 verwenden, für L1, L2 und L3 nicht funktioniert?

Ihrer Analyse fehlen zwei Schlüsselelemente - die Schwerkraft und wie diese Kraft (und die Zentrifugalkraft) mit r variieren. Die brauchst du. Gehen Sie in den rotierenden Rahmen um den Massenmittelpunkt des Systems und geben Sie die Schwerkraft beider Objekte sowie die Zentrifugalkraft ein (sie ist dort, weil wir uns in einem rotierenden Bezugsrahmen befinden). Sie sollten feststellen, dass sich alle diese Kräfte an den Punkten L1 und L3 zu Null addieren. Beachten Sie dann, dass die Kraft von den Punkten L1 und L3 weg zeigt, wenn Sie ein wenig variieren. Wenn Sie nicht über die mathematischen Fähigkeiten verfügen, um dies zu sehen, können Sie es einfach sagen - ein Objekt direkt bei L1 oder L3 kann mit derselben Umlaufzeit wie die umlaufenden Gravitationsquellen herumlaufen, aber jede Abweichung davon das verdirbt das. Das haben Sie gesagt, aber Sie haben nie die beteiligten Kräfte angegeben, Es reicht also nicht zu zeigen, dass es instabil ist, da die Instabilität von diesen Kräften abhängt. Wie sehr Sie das zeigen oder einfach sagen wollen, hängt von der Tiefe Ihrer Präsentation ab.